Đến nội dung

lahantaithe99 nội dung

Có 878 mục bởi lahantaithe99 (Tìm giới hạn từ 26-05-2020)



Sắp theo                Sắp xếp  

#504121 Ảnh thành viên

Đã gửi bởi lahantaithe99 on 05-06-2014 - 08:58 trong Góc giao lưu

attachicon.gifa.PNG

 

Mình là thằng mặc áo lạnh, người còn lại là tuananh2000 (http://diendantoanho...51-tuananh2000/) :))

Chú Phúc nhìn đệp trai lai láng thế :v  :wub:  :wub:




#493321 Ảnh thành viên

Đã gửi bởi lahantaithe99 on 16-04-2014 - 18:25 trong Góc giao lưu

Tôi nhìn thấy có 2 bạn gái rất xinh  >:)




#487497 Ảnh thành viên

Đã gửi bởi lahantaithe99 on 17-03-2014 - 21:50 trong Góc giao lưu

mũ đỏ " BOY" đấy

  :icon6:




#504351 Ảnh thành viên

Đã gửi bởi lahantaithe99 on 06-06-2014 - 07:16 trong Góc giao lưu

chú Tế cũng post hình đi chứ

Khổ nỗi anh chả có cái ảnh nào, toàn ảnh thẻ nhìn như thằng trốn tù thôi  :blush:  :ph34r:




#493328 Ảnh thành viên

Đã gửi bởi lahantaithe99 on 16-04-2014 - 19:18 trong Góc giao lưu

chả hiểu j cả. Người nào hàng mấy

Thì 2 bạn đứng gần nhau bên tay phải (so vs người xem), mà có 1 bạn giơ tay chỉ số 2 đó, bạn còn lại đứng ngay bên phải so vs bạn kia




#523556 Ảnh thành viên

Đã gửi bởi lahantaithe99 on 08-09-2014 - 21:59 trong Góc giao lưu

này đây :( liệu có được 10 like không ae?

 

1526609_421457907984123_1314193531_n.jpg

Nhìn anh Són mang 1 vẻ đẹp tiềm ẩn và thánh thiện  :closedeyes:  :closedeyes:  >:)  >:)  >:)  >:)  >:)  >:)




#493326 Ảnh thành viên

Đã gửi bởi lahantaithe99 on 16-04-2014 - 19:12 trong Góc giao lưu

where? chỉ tui  :ukliam2:  :ukliam2:

Hai bạn gái đứng gần nhất người xem ảnh  :icon6:




#487485 Ảnh thành viên

Đã gửi bởi lahantaithe99 on 17-03-2014 - 21:07 trong Góc giao lưu

:icon6:




#486960 Topic về Bất đẳng thức, cực trị THCS

Đã gửi bởi lahantaithe99 on 15-03-2014 - 17:07 trong Bất đẳng thức và cực trị

Cho các số dương a, b, c thỏa mãn a+ b2 + c2 =3

CMR: $\frac{a}{\sqrt{b}}$ + $\frac{b}{\sqrt{c}}$ + $\frac{c}{\sqrt{a}}$  $\geq$  a + b + c

Bài này bạn đã đăng [r bên này rồi mà!

Cách khác

http://diendantoanho...s-cao-xuân-huy/




#483339 Topic về Bất đẳng thức, cực trị THCS

Đã gửi bởi lahantaithe99 on 15-02-2014 - 22:28 trong Bất đẳng thức và cực trị

1)$Cho$$x+y+xy$$=24$. Tìm GTNN$x^{2}+y^{2}$

2)$Cho$$x^2+y^2-xy=4$. TÌm GTLN và GTNN của$x^2+y^2$

1.

$x+y+xy=24\leq x+y+\frac{(x+y)^2}{4}\Rightarrow x+y\geq 8$

$\Rightarrow x^2+y^2\geq \frac{(x+y)^2}{2}\geq \frac{8^2}{2}=32$

2.

$x^2+y^2-xy=4\geq 2xy-xy=xy$

$\Rightarrow x^2+y^2=4+xy\leq 4+4=8$




#504525 $\boxed{\text{Chuyên Đề}}$ Bất đẳng thức - Cực trị

Đã gửi bởi lahantaithe99 on 06-06-2014 - 19:32 trong Bất đẳng thức và cực trị

 

BÀI 176:

Cho a,b,c>0 thỏa $a^{2}+b^{2}+c^{2}=1$ . CM

$\frac{1}{1-bc}+\frac{1}{1-ac}+\frac{1}{1-ab}\leq \frac{9}{2}$

 

BĐT cần chứng minh $\Leftrightarrow \sum \frac{2bc}{2-2bc}\leqslant \frac{3}{2}$

 

Ta có 

 

$\sum \frac{2bc}{2-2bc}\leqslant \sum \frac{2bc}{2-(b^2+c^2)}=\sum \frac{2bc}{(a^2+b^2)+(a^2+c^2)}$

 

$=\frac{1}{2}\sum \frac{4bc}{(a^2+b^2)+(a^2+c^2)}\leqslant \frac{1}{2}\sum \frac{(b+c)^2}{(a^2+b^2)+(a^2+c^2)}$

 

$\leqslant \frac{1}{2}\sum (\frac{b^2}{a^2+b^2}+\frac{c^2}{a^2+c^2})=\frac{3}{2}$

 

Nên ta có đpcm




#493520 $\boxed{\text{Chuyên Đề}}$ Bất đẳng thức - Cực trị

Đã gửi bởi lahantaithe99 on 17-04-2014 - 17:05 trong Bất đẳng thức và cực trị

$144$,(Tự sáng tác ^^) Với $a,b,c>0$ tmđk $3a+5b+8c=1$

CMR; $(1-a)^{3}(1-b)^{5}(1-c)^{8} \geqslant 15^{16}a^{3}b^{5}c^{8}$ 

 

@Viet Hoang 99: Chú ý không kẹp $$ vào trong tiếng Việt có dấu.

 

Bài 141: Từ $abc=1\Rightarrow b=\frac{1}{ac}$

BĐT cần chứng minh $\Leftrightarrow a+c+\frac{1}{ac}\leq \left (a +c \right )\frac{1}{ac}+ac\Leftrightarrow \left ( 1-b \right )\left ( 1-a \right )\left ( 1-c \right )\geq 0$

Cái BĐT này luôn đúng..

Bài này sai đề, chắc anh Phuong Thu Quoc có nhầm lẫn gì  đó.

Vd như $a=\frac{1}{2};b=\frac{1}{4};c=8$ thì BĐT đâu có đúng?




#494364 $\boxed{\text{Chuyên Đề}}$ Bất đẳng thức - Cực trị

Đã gửi bởi lahantaithe99 on 21-04-2014 - 17:35 trong Bất đẳng thức và cực trị

Đóng góp 2 bài vậy !! :P

Bài 150 : Cho $a;b;c$ là các số thực dương. Tìm $GTNN$ của : 
$$Q=\sum \sqrt[4]{\frac{a}{b+c}}+\sum \sqrt{\frac{b+c}{a}}$$
 

Bài này nhìn số má trâu bò quá!  :wacko:  :wacko:  :wacko:

$150/$

Áp dụng BĐT Cauchy ta có

 

$Q\geqslant 3\left [ \sqrt[12]{\frac{abc}{(a+b)(b+c)(c+a)}} +\sqrt[6]{\frac{(a+b)(b+c)(c+a)}{abc}}\right ]$

 

$=3\left [ \sqrt[12]{\frac{abc}{(a+b)(b+c)(c+a)}}+\frac{1}{\sqrt[4]{8}}.\sqrt[6]{\frac{(a+b)(b+c)(c+a)}{abc}} \right ]+3(1-\frac{1}{\sqrt[4]{8}})\sqrt[6]{\frac{(a+b)(b+c)(c+a)}{abc}}$

 

 Cô si cho biểu thức thứ nhất

 

Biếu thức $(1)$ $\geqslant 2\sqrt[24]{\frac{(a+b)(b+c)(c+a)}{512abc}}\geqslant 2.\sqrt[24]{\frac{1}{64}}=2.\sqrt[4]{\frac{1}{2}}$

 

Biểu thức $(2)$ $(1-\frac{1}{\sqrt[4]{8}})\sqrt[6]{\frac{(a+b)(b+c)(c+a)}{abc}}\geqslant (1-\frac{1}{\sqrt[4]{8}})\sqrt[6]{8}=\sqrt{2}-\sqrt[4]{\frac{1}{2}}$

 

Cộng vế suy ra $Q\geqslant 3(\sqrt[4]{\frac{1}{2}}+\sqrt{2})$

 

P/s: bài này cồng kềnh tốn bao nhiêu t/g, mà lại còn k biết có đúng k nữa 




#495498 $\boxed{\text{Chuyên Đề}}$ Bất đẳng thức - Cực trị

Đã gửi bởi lahantaithe99 on 27-04-2014 - 17:51 trong Bất đẳng thức và cực trị

 

Tương tự : 

$\frac{1}{b+c+1} \le \dfrac{\sqrt[3]{a}}{\sqrt[3]{a}+\sqrt[3]{b}+\sqrt[3]{c} }$

$\frac{1}{c+a+1} \le \dfrac{\sqrt[3]{b}}{\sqrt[3]{a}+\sqrt[3]{b}+\sqrt[3]{c} }$

$\Rightarrow \frac{1}{a+b+1} +\frac{1}{b+c+1}+\frac{1}{c+a+1}\leq 1$

Ta cần chứng minh $\frac{1}{2+a}+\frac{1}{2+b}+\frac{1}{2+c}\geq 1$

Thật vậy, áp dụng bdt $Cauchy$, ta có :

$\frac{1}{2+a}+\frac{2+a}{9}\geq \frac{2}{3}$

$\frac{1}{2+b}+\frac{2+b}{9}\geq \frac{2}{3}$

$\frac{1}{2+c}+\frac{2+c}{9}\geq \frac{2}{3}$

$\Rightarrow \frac{1}{2+a}+ \frac{1}{2+b}+ \frac{1}{2+c}\geq 2-\frac{6+a+b+c}{9}$
                                                     $  \geq 2-\frac{6+3\sqrt[3]{abc}}{9}\geq 1$
$\Rightarrow dpcm$
Dấu ''$=$'' xảy ra $\Leftrightarrow  a=b=c=1$
 
p.s: bạn xem có sai chỗ nào không ? 

 

Ngược dấu rồi anh ơi




#489815 $\boxed{\text{Chuyên Đề}}$ Bất đẳng thức - Cực trị

Đã gửi bởi lahantaithe99 on 31-03-2014 - 17:43 trong Bất đẳng thức và cực trị

Theo hệ thức Viet thì $x_{1}.x_{2}=1\Rightarrow x_{1}=\frac{1}{x_{2}}$

 

Áp dụng BĐT Cô si thì 

 

$T=x_{1}^4+x_{2}^2=\frac{1}{x_{2}^4}+x_{2}^2=\frac{1}{x_{2}^4}+\frac{x_{2}^2}{2}+\frac{x_{2}^2}{2}\geqslant 3.\sqrt[3]{\frac{1}{4}}$




#486315 $\boxed{\text{Chuyên Đề}}$ Bất đẳng thức - Cực trị

Đã gửi bởi lahantaithe99 on 08-03-2014 - 21:05 trong Bất đẳng thức và cực trị

sai chỗ màu đỏ này:$b\leq ...;b\leq ...$

Chỗ màu xanh : $b\leq ...$ nhưng chắc gì $a<\frac{3}{2}\Rightarrow b<\frac{3}{2}$

Cậu ko đọc kỹ đề bài sao. ĐỀ BÀI CHO $b<a$ rồi mà  ~O)




#486118 $\boxed{\text{Chuyên Đề}}$ Bất đẳng thức - Cực trị

Đã gửi bởi lahantaithe99 on 07-03-2014 - 12:36 trong Bất đẳng thức và cực trị

Mình xin đóng góp ít bài :icon6: .

 

109) Cho a,b,c>0 t/m: $a+b+c\geq \frac{1}{a}+\frac{1}{b}+\frac{1}{c}$.

CMR: $a+b+c\geq \frac{3}{a+b+c}+\frac{2}{abc}$

 

Từ giả thiết dễ dàng suy ra $a+b+c\geq 3\Leftrightarrow \frac{a+b+c}{3}\geq \frac{3}{a+b+c}(1)$

$GT\Leftrightarrow abc(a+b+c)\geq ab+bc+ac\Rightarrow \frac{2}{abc}\leq \frac{2(a+b+c)}{ab+bc+ac}$

Lại có $\frac{(ab+bc+ac)^2}{3}\geq abc(a+b+c)\geq ab+bc+ac\Rightarrow ab+bc+ac\geq 3$

$\Rightarrow \frac{2}{abc}\leq \frac{2(a+b+c)}{ab+bc+ac}\leq \frac{2(a+b+c)}{3}(2)$

$(1);(2)$ suy ra đpcm




#484933 $\boxed{\text{Chuyên Đề}}$ Bất đẳng thức - Cực trị

Đã gửi bởi lahantaithe99 on 26-02-2014 - 20:32 trong Bất đẳng thức và cực trị

96/ Cho $0<x;y;z \leq \frac{3}{2}$. Chứng minh rằng $\sum \sqrt{x^2+\frac{1}{x^2}} \geq \frac{3}{2}.\sqrt{17}$

 

 

Áp dụng bđt Bunhia

 

 

$\sqrt{(x^2+\frac{1}{x^2})(1+4^2)}\geq x+\frac{4}{x}$

 

$\Rightarrow \sqrt{17}.\sum \sqrt{x^2+\frac{1}{x^2}}\geq\sum x+\sum \frac{4}{x}$

 

$\geq\sum x+\sum \frac{1}{4x}+\sum \frac{15}{4x}$

 

$\geq 3+\sum \frac{15}{4x}$ (áp dụng cô si)

 

$\geq 3+\frac{15.9}{4x+4y+4z}\geq 3+\frac{45}{2}=\frac{51}{2}$

 

$\Rightarrow \sum \sqrt{x^2+\frac{1}{x^2}}\geq \frac{51}{2\sqrt{17}}=\frac{3}{2}.\sqrt{17}$




#482617 $\boxed{\text{Chuyên Đề}}$ Bất đẳng thức - Cực trị

Đã gửi bởi lahantaithe99 on 11-02-2014 - 21:48 trong Bất đẳng thức và cực trị

 

31) Cho $\left\{\begin{matrix}a_1;a_2;...;a_n>0 & & \\ a_1+a_2+...+a_n=1 & & \end{matrix}\right.$. Cmr: $(\frac{1}{a_1}-1)(\frac{1}{a_2}-1)...(\frac{1}{a_n}-1)\geq (n-1)^n$

 

 

 

Ta có $a_{1}+...+a_{n}=1\Rightarrow 1-a_{1}=a_{2}+...+a_{n}\geq (n-1).\sqrt[n-1]{a_{2}...a_{n}}$
Tươn tự $1-a_{n}\geq (n-1).\sqrt[n-1]{a_{1}...a_{n-1}}$
$\Rightarrow \prod (1-a_{1})\geq (n-1)^2.a_{1}...a_{n}$
$\Rightarrow \prod (\frac{1}{a_{1}}-1)\geq (n-1)^2$



#487348 $\boxed{\text{Chuyên Đề}}$ Bất đẳng thức - Cực trị

Đã gửi bởi lahantaithe99 on 17-03-2014 - 13:50 trong Bất đẳng thức và cực trị

Loạn hết rồi à? @@
 

106) Cho $a;b;c>0$ thỏa: $a^2+b^2+c^2=1$. Cmr: $\sum \frac{1}{a^2+b^2}\leq \frac{\sum a^3}{2abc}+3$

 

107)

a) Cho $x;y>0$. Cmr: $\sum \frac{1}{x^2}\geq \frac{8}{(x+y)^2}$

b) Cho $a;b;c>0$ thỏa: $ab+bc+ca\leq abc$. Cmr: $\sum \frac{8}{a+b}\leq \sum \frac{b+c}{a^2}+2$

 

 

 

 

106

Áp dụng bđt Co si $b^2+c^2\geqslant 2bc\Rightarrow \frac{a^2}{b^2+c^2}\leqslant \frac{a^2}{2bc}$

 

$\Rightarrow \sum \frac{a^2}{b^2+c^2}\leqslant \sum \frac{a^2}{2bc}=\frac{a^3+b^3+c^3}{2abc}$

 

$\Rightarrow \sum \frac{a^2+b^2+c^2}{b^2+c^2}\leqslant \frac{a^3+b^3+c^3}{2abc}+3$

$\Leftrightarrow \sum \frac{1}{b^2+c^2}\leqslant \frac{a^3+b^3+c^3}{2abc}+3$

Bài 107

a

$\sum \frac{1}{x^2}\geqslant \frac{1}{2}(\frac{1}{x}+\frac{1}{y})^2=\frac{1}{2}.\frac{(x+y)^2}{x^2y^2}$ $(1)$

 

Theo bdt Co si $xy\leqslant \frac{(x+y)^2}{4}\Rightarrow x^2y^2\leqslant \frac{(x+y)^4}{16}$ $(2)$

 

$(1);(2)\Rightarrow \sum \frac{1}{x^2}\geqslant \frac{8}{(x+y)^2}$

b) Từ phần a suy ra

 

$\frac{a+b}{a^2}+\frac{a+b}{b^2}\geqslant \frac{8}{a+b}$

 

Do đó

 

$\sum \frac{8}{a+b}\leqslant \frac{a+b}{a^2}+\frac{a+b}{b^2}+\frac{b+c}{b^2}+\frac{b+c}{c^2}+\frac{a+c}{a^2}+\frac{a+c}{c^2}$

 

$=\frac{a}{b^2}+\frac{b}{a^2}+\frac{c}{b^2}+\frac{b}{c^2}+\frac{c}{a^2}+\frac{a}{c^2}+2(\frac{1}{a}+\frac{1}{b}+\frac{1}{c})$

 

$=\frac{a+c}{b^2}+\frac{b+c}{a^2}+\frac{a+b}{c^2}+2(\frac{1}{a}+\frac{1}{b}+\frac{1}{c})$ $(1)$

 

Từ $GT :ab+bc+ac\leqslant abc\Rightarrow \frac{1}{a}+\frac{1}{b}+\frac{1}{c}\leqslant 1\Rightarrow 2\sum\frac{1}{a}\leqslant 2$ $(2)$

 

Từ $(1);(2)$ ta đc đpcm

c)

Áp dụng bđt $AM-GM$

$\frac{1}{(a-b)^2}+\frac{1}{(b-c)^2}\geqslant \frac{2}{(a-b)(b-c)}\geqslant \frac{8}{(a-c)^2}$

$\Rightarrow VT\geqslant \frac{a^2+c^2}{2}+\frac{8}{(a-c)^2}$

$=\frac{2ac}{2}+\frac{(a-c)^2}{2}+\frac{8}{(a-c)^2}\geqslant ac+4$ (cô si)




#488063 $\boxed{\text{Chuyên Đề}}$ Bất đẳng thức - Cực trị

Đã gửi bởi lahantaithe99 on 21-03-2014 - 12:38 trong Bất đẳng thức và cực trị

Cho 3 số dương a, b, c thoả mãn a + b + c = 1

a, CMR $\frac{a + bc}{b + c}$  +  $\frac{b + ca}{c + a}$  +  $\frac{c + ab}{a + b}$    $\geq$   2

b, Tìm GTNN của P = $\frac{a}{ab + c}$  +  $\frac{b}{bc + a}$ + $\frac{c}{ac + b}$

 

P/s:Bài này sử dụng phương pháp đổi biến nhé!!!

a)

 

Ta có $\sum \frac{a+bc}{b+c}=\sum \frac{a(a+b+c)+bc}{b+c}=\sum \frac{(a+b)(a+c)}{b+c}$

 

Cô si từng cặp một ta có 

 

$\sum \frac{(a+b)(a+c)}{b+c}\geqslant 2(a+b+c)=2$

 

b) 

Tương tự phần a ta có

 

$\sum \frac{a}{ab+c}=\sum \frac{a}{(c+a)(c+b)}$

 

Áp dụng bđt Cauchy

 

$\frac{a}{(a+c)(c+b)}+\frac{27a(b+c)}{8}+\frac{27a(a+c)}{8}\geqslant \frac{27}{4}a$

 

Thiết lập tương tự ta có

 

$\sum \frac{a}{(a+c)(c+b)}+\frac{27}{8}[(a+b+c)^2+ab+bc+ac]\geqslant \frac{27}{4}\sum a$

 

$\Leftrightarrow \sum \frac{a}{(a+c)(c+b)}\geqslant \frac{27}{4}-\frac{27}{8}-\frac{27}{8}(ab+bc+ac)$

 

Lại có $ab+bc+ac\leqslant \frac{1}{3}$

 

$\Rightarrow \sum \frac{a}{(b+c)(a+c)}\geqslant \frac{9}{4}$




#490839 $\boxed{\text{Chuyên Đề}}$ Bất đẳng thức - Cực trị

Đã gửi bởi lahantaithe99 on 05-04-2014 - 19:12 trong Bất đẳng thức và cực trị

Bài 148. Cho $a,b,c>0.$ Tìm $maxP=\frac{1}{\sqrt{a^2+b^2+c^2}}-\frac{2}{(a+1)(b+1)(c+1)}$

Bài này theo mình không tìm đc max (ý kiến cá nhân) 

Thấy tính đối xứng của $a,b,c$ và $a,b,c>0$ nên chác dấu $=$ xảy ra khi $a=b=c$

Thay $a=b=c$ bằng một số giá trị thì cứ giá trị càng nhỏ thì $P$ càng lớn, Vd như $a=b=c=0,0001$ thì $P$ lên đến hơn $5000$, vậy nếu $a=b=c$ bàng một số nhỏ hơn nữa thì không biết giá trị là bao nhiêu???????????




#482613 $\boxed{\text{Chuyên Đề}}$ Bất đẳng thức - Cực trị

Đã gửi bởi lahantaithe99 on 11-02-2014 - 21:39 trong Bất đẳng thức và cực trị



 

 

32) Cho $x+y+z=0$. Tìm Min $P=\sum \sqrt{3+4^x}$

 

 

Ta có $\sqrt{(3+4^x)(3+1)}\geq (3+\sqrt{4^x})\Rightarrow \sqrt{3+4^x}\geq \frac{3+\sqrt{4^x}}{2}$

$\Rightarrow \sum \sqrt{3+4^x}\geq \frac{9+\sum \sqrt{4^x}}{2}$

$\geq \frac{9+3.\sqrt[3]{4^x.4^y.4^z}}{2}=\frac{9+3.\sqrt[3]{4^{x+y+z}}}{2}=6$




#488588 $\boxed{\text{Chuyên Đề}}$ Bất đẳng thức - Cực trị

Đã gửi bởi lahantaithe99 on 24-03-2014 - 19:53 trong Bất đẳng thức và cực trị

TOPIC yêu cầu bài nào đã làm xong được tô màu đỏ  :angry: 

 

119) Cho tam giác $ABC$ có diện tích là $\frac{3}{2}$. CMR: $(\frac{1}{a}+\frac{1}{b}+\frac{1}{c})(\frac{1}{h_{a}}+\frac{1}{h_{b}}+\frac{1}{h_{c}})\geq 3$

 

 

 

 

119 Ta có 

 

$a.h_{a}=b.h_{b}=c.h_{c}=3\Rightarrow a=\frac{h_{a}}{3};b=\frac{h_{b}}{3};c=\frac{h_{c}}{3}$

 

Khi đó

 

$(\sum \frac{1}{a})(\sum\frac{1}{h_{a}} )=(\sum \frac{h_{a}}{3})(\sum \frac{1}{h_{a}})\geqslant (\frac{3}{\sqrt{3}})^2=3$

 

(đúng theo bđt Bunhiacopxki




#492301 $\boxed{\text{Chuyên Đề}}$ Bất đẳng thức - Cực trị

Đã gửi bởi lahantaithe99 on 11-04-2014 - 22:12 trong Bất đẳng thức và cực trị

Bài 140.  Cho $a,b,c,d,e$ là các số thực không âm biết $a+b+c+d+e=5$. Chứng minh : $abc+bcd+cde+dea+eab\leq 5$

Ở đây này

http://diendantoanho...từ-mathlinksro/